Difference between revisions of "2021 Fall AMC 10B Problems/Problem 17"

m (Solution 1)
Line 4: Line 4:
 
<math>(\textbf{A})\: 5x+2y=0\qquad(\textbf{B}) \: 3x+2y=0\qquad(\textbf{C}) \: x-3y=0\qquad(\textbf{D}) \: 2x-3y=0\qquad(\textbf{E}) \: 5x-3y=0</math>
 
<math>(\textbf{A})\: 5x+2y=0\qquad(\textbf{B}) \: 3x+2y=0\qquad(\textbf{C}) \: x-3y=0\qquad(\textbf{D}) \: 2x-3y=0\qquad(\textbf{E}) \: 5x-3y=0</math>
  
 +
==Solution 1==
 +
Denote <math>O</math> as the origin.
 +
 +
Even though the problem is phrased as a coordinate bash, that looks disgusting. Instead, let's try to phrase this problem in terms of Euclidean geometry, using the observation that <math>\angle POP'' = 90^{\circ}</math>, and that both <math>\ell</math> and <math>m</math> must pass through <math>O</math> in order to preserve the distance from <math>P</math> to the origin.
 +
<asy>
 +
unitsize(1.4cm);
 +
draw((0,3)--(0,0)--(3,0), dashed);
 +
dot((0,3));
 +
dot((3,0));
 +
label("$P$", (0,3), W);
 +
label("$P''$", (3,0), S);
 +
 +
draw((0,0)--(1.5,4.5));
 +
label("$\ell$", (1.5,4.5), N);
 +
draw((0,0)--(4,2));
 +
label("$m$", (4,2), E);
 +
 +
dot((1.8,2.4));
 +
label("$P'$", (1.8,2.4), N);
 +
label("$O$",(0,0));
  
==Solution 1==
+
dot((1,3)); dot((2.5,1.25));
 +
label("$A$", (1,3), E); label("$B$", (2.5,1.25), N);
 +
</asy>
 +
(<math>A</math> and <math>B</math> are just defined as points on lines <math>\ell</math> and <math>m</math>.)
 +
Because of how reflections work, we have that <math>\angle AOP' = \angle POA</math> and <math>\angle P'OB = \angle BOP''</math>; adding these two equations together and using angle addition, we have that <math>\angle AOB = \angle POA + \angle BOP''</math>. Since the sum of both sides combined must be <math>90^{\circ}</math> by angle addition,
 +
<cmath>\angle AOB = 45^{\circ}.</cmath>
 +
This is helpful! We can now return to using coordinates, with this piece of information in mind:
 +
<asy>
 +
unitsize(0.2cm);
 +
markscalefactor = 0.08;
 +
import graph;
 +
Label f;
 +
f.p=fontsize(9);
 +
xaxis(-2,6,Ticks(f, 2.0));
 +
yaxis(-1,6,Ticks(f, 2.0));
 +
dot((-1,4));
 +
label("$P$", (-1,4), W);
 +
dot((4,1));
 +
label("$P''$", (4,1), W);
 +
 
 +
draw((0,0)--(1.2,6));
 +
label("$\ell$", (1.2,6), N);
 +
dot((0.5,2.5));label("$(0.5,2.5)$", (0.5,2.5), E);label("$A$", (0.5,2.5), W);
 +
dot((3,2));label("$B$", (3,2), E);
 +
draw((0.5,2.5)--(3,2), dashed);
 +
 
 +
draw((0,0)--(6,4));
 +
label("$m$", (6,4), E);
 +
 
 +
draw(anglemark((6, 4), (0, 0), (1, 5)));
 +
label("$45^{\circ}$", (0.54,0.75));
 +
</asy>
 +
The <math>45^{\circ}</math> angle is a little bit unwieldy in the coordinate plane. To fix this, let's make a <math>45-45-90</math> triangle. Let <math>A</math> be a point on <math>\ell</math>; to make <math>A</math> fit nicely in the diagram, let it be <math>(0.5,2.5)</math>. Now, let's draw a perpendicular to <math>\ell</math> through point <math>A</math>, intersecting <math>m</math> at point <math>B</math>. <math>OAB</math> is a <math>45-45-90</math> triangle, so <math>B</math> is a <math>90</math> degree counterclockwise rotation from <math>O</math> about <math>A</math>. Therefore, the coordinates of <math>B</math> are
 +
<cmath>(0.5+2.5,2.5-0.5) = (3,2).</cmath>
 +
So, <math>(3,2)</math> is a point on line <math>m</math>, which we already know passes through the origin; therefore, <math>m</math>'s equation is <math>y=\frac{2x}{3} \implies \boxed{\textbf{(D) } 2x-3y = 0}.</math>
 +
~ihatemath123
 +
 
 +
==Solution 2==
 
It is well known that the composition of 2 reflections , one after another, about two lines <math>l</math> and <math>m</math>, respectively, that meet at an angle <math>\theta</math> is a rotation by <math>2\theta</math> around the intersection of <math>l</math> and <math>m</math>.  
 
It is well known that the composition of 2 reflections , one after another, about two lines <math>l</math> and <math>m</math>, respectively, that meet at an angle <math>\theta</math> is a rotation by <math>2\theta</math> around the intersection of <math>l</math> and <math>m</math>.  
  
Line 14: Line 71:
 
~hurdler, minor edits by nightshade2526
 
~hurdler, minor edits by nightshade2526
  
==Solution 2==
+
==Solution 3==
  
 
We know that the equation of line <math>\ell</math> is <math>y = 5x</math>. This means that <math>P'</math> is <math>(-1,4)</math> reflected over the line <math>y = 5x</math>. This means that the line with <math>P</math> and <math>P'</math> is perpendicular to <math>\ell</math>, so it has slope <math>-\frac{1}{5}</math>. Then the equation of this perpendicular line is <math>y = -\frac{1}{5}x + c</math>, and plugging in <math>(-1,4)</math> for <math>x</math> and <math>y</math> yields <math>c = \frac{19}{5}</math>.  
 
We know that the equation of line <math>\ell</math> is <math>y = 5x</math>. This means that <math>P'</math> is <math>(-1,4)</math> reflected over the line <math>y = 5x</math>. This means that the line with <math>P</math> and <math>P'</math> is perpendicular to <math>\ell</math>, so it has slope <math>-\frac{1}{5}</math>. Then the equation of this perpendicular line is <math>y = -\frac{1}{5}x + c</math>, and plugging in <math>(-1,4)</math> for <math>x</math> and <math>y</math> yields <math>c = \frac{19}{5}</math>.  

Revision as of 12:16, 9 April 2022

Problem

Distinct lines $\ell$ and $m$ lie in the $xy$-plane. They intersect at the origin. Point $P(-1, 4)$ is reflected about line $\ell$ to point $P'$, and then $P'$ is reflected about line $m$ to point $P''$. The equation of line $\ell$ is $5x - y = 0$, and the coordinates of $P''$ are $(4,1)$. What is the equation of line $m?$

$(\textbf{A})\: 5x+2y=0\qquad(\textbf{B}) \: 3x+2y=0\qquad(\textbf{C}) \: x-3y=0\qquad(\textbf{D}) \: 2x-3y=0\qquad(\textbf{E}) \: 5x-3y=0$

Solution 1

Denote $O$ as the origin.

Even though the problem is phrased as a coordinate bash, that looks disgusting. Instead, let's try to phrase this problem in terms of Euclidean geometry, using the observation that $\angle POP'' = 90^{\circ}$, and that both $\ell$ and $m$ must pass through $O$ in order to preserve the distance from $P$ to the origin. [asy] unitsize(1.4cm); draw((0,3)--(0,0)--(3,0), dashed); dot((0,3)); dot((3,0)); label("$P$", (0,3), W); label("$P''$", (3,0), S);  draw((0,0)--(1.5,4.5)); label("$\ell$", (1.5,4.5), N); draw((0,0)--(4,2)); label("$m$", (4,2), E);  dot((1.8,2.4)); label("$P'$", (1.8,2.4), N); label("$O$",(0,0));  dot((1,3)); dot((2.5,1.25)); label("$A$", (1,3), E); label("$B$", (2.5,1.25), N); [/asy] ($A$ and $B$ are just defined as points on lines $\ell$ and $m$.) Because of how reflections work, we have that $\angle AOP' = \angle POA$ and $\angle P'OB = \angle BOP''$; adding these two equations together and using angle addition, we have that $\angle AOB = \angle POA + \angle BOP''$. Since the sum of both sides combined must be $90^{\circ}$ by angle addition, \[\angle AOB = 45^{\circ}.\] This is helpful! We can now return to using coordinates, with this piece of information in mind: [asy] unitsize(0.2cm); markscalefactor = 0.08; import graph; Label f;  f.p=fontsize(9);  xaxis(-2,6,Ticks(f, 2.0));  yaxis(-1,6,Ticks(f, 2.0)); dot((-1,4)); label("$P$", (-1,4), W); dot((4,1)); label("$P''$", (4,1), W);  draw((0,0)--(1.2,6)); label("$\ell$", (1.2,6), N); dot((0.5,2.5));label("$(0.5,2.5)$", (0.5,2.5), E);label("$A$", (0.5,2.5), W); dot((3,2));label("$B$", (3,2), E); draw((0.5,2.5)--(3,2), dashed);  draw((0,0)--(6,4)); label("$m$", (6,4), E);  draw(anglemark((6, 4), (0, 0), (1, 5))); label("$45^{\circ}$", (0.54,0.75)); [/asy] The $45^{\circ}$ angle is a little bit unwieldy in the coordinate plane. To fix this, let's make a $45-45-90$ triangle. Let $A$ be a point on $\ell$; to make $A$ fit nicely in the diagram, let it be $(0.5,2.5)$. Now, let's draw a perpendicular to $\ell$ through point $A$, intersecting $m$ at point $B$. $OAB$ is a $45-45-90$ triangle, so $B$ is a $90$ degree counterclockwise rotation from $O$ about $A$. Therefore, the coordinates of $B$ are \[(0.5+2.5,2.5-0.5) = (3,2).\] So, $(3,2)$ is a point on line $m$, which we already know passes through the origin; therefore, $m$'s equation is $y=\frac{2x}{3} \implies \boxed{\textbf{(D) } 2x-3y = 0}.$ ~ihatemath123

Solution 2

It is well known that the composition of 2 reflections , one after another, about two lines $l$ and $m$, respectively, that meet at an angle $\theta$ is a rotation by $2\theta$ around the intersection of $l$ and $m$.

Now, we note that $(4,1)$ is a 90 degree rotation clockwise of $(-1,4)$ about the origin, which is also where $l$ and $m$ intersect. So $m$ is a 45 degree rotation of $l$ about the origin clockwise.

To rotate $l$ 90 degrees clockwise, we build a square with adjacent vertices $(0,0)$ and $(1,5)$. The other two vertices are at $(5,-1)$ and $(6,4)$. The center of the square is at $(3,2)$, which is the midpoint of $(1,5)$ and $(5,-1)$. The line $m$ passes through the origin and the center of the square we built, namely at $(0,0)$ and $(3,2)$. Thus the line is $y = \frac{2}{3} x$. The answer is (D) $\boxed{2x - 3y = 0}$.

~hurdler, minor edits by nightshade2526

Solution 3

We know that the equation of line $\ell$ is $y = 5x$. This means that $P'$ is $(-1,4)$ reflected over the line $y = 5x$. This means that the line with $P$ and $P'$ is perpendicular to $\ell$, so it has slope $-\frac{1}{5}$. Then the equation of this perpendicular line is $y = -\frac{1}{5}x + c$, and plugging in $(-1,4)$ for $x$ and $y$ yields $c = \frac{19}{5}$.

The midpoint of $P'$ and $P$ lies at the intersection of $y = 5x$ and $y = -\frac{1}{5}x + \frac{19}{5}$. Solving, we get the x-value of the intersection is $\frac{19}{26}$ and the y-value is $\frac{95}{26}$. Let the x-value of $P'$ be $x'$ - then by the midpoint formula, $\frac{x' - 1}{2} = \frac{19}{26} \implies x' = \frac{32}{13}$. We can find the y-value of $P'$ the same way, so $P' = (\frac{32}{13},\frac{43}{13})$.

Now we have to reflect $P'$ over $m$ to get to $(4,1)$. The midpoint of $P'$ and $P''$ will lie on $m$, and this midpoint is, by the midpoint formula, $(\frac{42}{13},\frac{28}{13})$. $y = mx$ must satisfy this point, so $m = \frac{\frac{28}{13}}{\frac{42}{13}} = \frac{28}{42} = \frac{2}{3}$.

Now the equation of line $m$ is $y = \frac{2}{3}x \implies 2x-3y = 0 = \boxed{D}$


~KingRavi

Video Solution

Solution 2021 Fall 10B #17

~hurdler

See Also

2021 Fall AMC 10B (ProblemsAnswer KeyResources)
Preceded by
Problem 16
Followed by
Problem 18
1 2 3 4 5 6 7 8 9 10 11 12 13 14 15 16 17 18 19 20 21 22 23 24 25
All AMC 10 Problems and Solutions

The problems on this page are copyrighted by the Mathematical Association of America's American Mathematics Competitions. AMC logo.png